-1

Motivation: (If you don't need the motivation, skip it.)

I've tried partitioning $\mathbb{R}$ into sets $A$ and $B$, where the Lebesgues measures in every non-empty open interval have a non-zero constant ratio not equal to one. It turns out such a constant doesn't exist.

I also tried partitioning $\mathbb{R}$ into sets $A$ and $B$, such that the measures of $A$ and $B$ in every non-empty open interval have an "almost" non-zero constant ratio. This is also impossible, since the lower bound of the non-constant ratio is always zero and the upper bound is positive infinity.

Hence, I made non-empty open intervals with a constant length, such that the measures of $A$ and $B$ in every non-empty open interval have a constant ratio w.r.t. the constant length.


Thereby, suppose $\lambda$ is the Lebesgue measure on the Borel $\sigma$-algebra.

Does there exist an example of sets $A,B\subset\mathbb{R}$, where:

  1. $A\cup B=\mathbb{R}$
  2. $A\cap B=\emptyset$
  3. For all $\varepsilon>0$, there exists a non-zero constant $c$, where for all $a\in\mathbb{R}$:

$$\lambda(A\cap (a,a+\varepsilon))=c\cdot\lambda(B \cap (a,a+\varepsilon))?$$

Arbuja
  • 53
  • 2
    It seems you messed up the intended order of the quantifiers. Did you really mean $c$ to depend on both $a$ and $\epsilon$, as written? And why do you quantify over $I$ when $I$ is uniquely determined by $a$ and $\epsilon$? – Emil Jeřábek Apr 22 '25 at 17:58
  • @EmilJeřábek I was hoping $A$ and $B$ can exist. What should be the order of the quantifiers. Note, I want people to know that $I$ is an open interval. – Arbuja Apr 22 '25 at 18:24
  • @EmilJeřábek I made edits. Is this better? – Arbuja Apr 22 '25 at 18:29
  • Just asking: Why in a measure theory context would it matter whether $I$ is open, closed, or half-open? – coffeemath Apr 22 '25 at 18:33
  • Every open interval is of the form $(a,a+\epsilon)$ for some $a$ and $\epsilon>0$. Hence what you have now is the same question as in https://math.stackexchange.com/questions/5042199/can-we-partiton-%E2%84%9D-into-sets-a-and-b-where-the-lebesgue-measures-in-every-no . – Emil Jeřábek Apr 22 '25 at 18:43
  • @EmilJeřábek In that case, I want $c$ to depend on both $a$ and $\epsilon$ – Arbuja Apr 22 '25 at 18:48
  • @coffeemath I recalled when I read this question, it required the use of a non-empty open interval. I’m not sure why this matters? – Arbuja Apr 22 '25 at 18:50
  • Then there is not much point in calling $c$ a constant. You are simply asking if there is a measurable subset $A\subseteq\mathbb R$ such that $0<\lambda(A\cap I)<\lambda I$ for every nondegenerate interval $I$ (the answer to which is not immediately clear to me). – Emil Jeřábek Apr 22 '25 at 18:59
  • 2
    Oh, actually, it is answered positively in https://mathoverflow.net/a/42121/ that you linked to. – Emil Jeřábek Apr 22 '25 at 19:07
  • @EmilJeřábek So, there’s nothing more to explore? (I’ll delete this question after your response.) – Arbuja Apr 22 '25 at 19:09

1 Answers1

2

If you arrange the quantifier in this way, i.e. the constant does not depend on $a$, then no such pair of sets can exist.

First thing to note is that for any open, nonempty interval $I$ we must have $\lambda(A\cap I), \lambda(B\cap I)>0$. If one of them was zero, then both are zero due to the requirement that $c>0$. However, they cannot both be zero as otherwise we would get, due to $A\cup B=\mathbb{R}$, that $$ 0<\lambda(I) = \lambda(I \cap A)+\lambda(I\cap B)=0. $$

For all $\delta\in [0,1]$ we have $$ c_{1+\delta} \lambda(B\cap (0,1+\delta)) = \lambda(A\cap (0,1+\delta)) \leq \lambda(A\cap (0,1)) + \lambda(A\cap(\delta, 1+\delta)) = c_1 (\lambda(B\cap (0,1))+\lambda(B\cap (\delta, 1+\delta))) \leq 2c_1 \lambda(B\cap (0,1+\delta)). $$

Thus, we get for all $\delta\in[0,1]$ that $$ c_{1+\delta} \leq 2c_1. $$

Furthermore, we get for all positive natural numbers $n$ that $$ c_1 \lambda(B\cap (0,1)) = \lambda(A\cap(0,1)) = \lambda(A\cap (0,1/n)) + \dots + \lambda(A\cap ((n-1)/n,1)) = c_{1/n} (\lambda(B\cap (0,1/n)) + \dots + \lambda(B\cap ((n-1)/n,1))) = c_{1/n}\lambda(B\cap (0,1)). $$ Thus, we get $c_1=c_{1/n}$. In fact, we get for all $\delta>0$ and all positive natural numbers $n$ that $$ c_{\delta} = c_{\delta/n}. $$ Combining everything, we obtain for all $\varepsilon>0$ we have $$ c_\varepsilon \leq 2c_1.$$ However, as the density is now bounded from above, the arguments in your previous question Partitioning ℝ into sets $A$ and $B$, such that the measures of $A$ and $B$ in each non-empty open interval have an "almost" non-zero constant ratio apply and rule out that such a pair of sets can exist.

  • Is there any way to arrange the quantifiers, where the sets exist? – Arbuja Apr 22 '25 at 20:00
  • 3
    The only nontrivial statement I could come up with is that there exists partition such that $\lambda(A\cap I), \lambda(B\cap I)>0$ for all non-empty, open intervals $I$. As was noted in the comments above, this is covered in https://mathoverflow.net/questions/42119/sets-with-equal-positive-measure-in-every-interval/42121#42121. It might be helpful to know why you need/want a partition with such properties. – Severin Schraven Apr 22 '25 at 20:06
  • I was hoping to apply a simple function, where the parititions of the graph of the function can have a measure that gets infinitsimally close to zero: e.g., $\mathbb{1}{A}(x)\sin(x)+\mathbb{1}{A}(x)$. This is averaged using Section 3.1 and 5.3 of this paper. The current example only applies to nowehere continuous functions defined on the rationals. (There might be a better example) – Arbuja Apr 22 '25 at 20:32
  • If you have any time (or energy) left, consider this post. I will award you a +100 bounty. – Arbuja Apr 23 '25 at 19:39
  • 1
    @Arbuja I will have a look, but I won't have much time in the next weeks. However, there loooots of more competent people on this site, so I don't think you'll need me – Severin Schraven Apr 23 '25 at 20:48